LSAT and Law School Admissions Forum

Get expert LSAT preparation and law school admissions advice from PowerScore Test Preparation.

User avatar
 Dave Killoran
PowerScore Staff
  • PowerScore Staff
  • Posts: 5852
  • Joined: Mar 25, 2011
|
#27068
Complete Question Explanation
(The complete setup for this game can be found here: lsat/viewtopic.php?t=8627)

The correct answer choice is (A)

The application of proper List question technique (take a single rule and apply it to all five answer choices consecutively; take another rule and apply it to the remaining answer choices, etc.) eliminates every answer except for answer choice (A). Answer choice (B) is incorrect since both P and L are reduced. Answer choice (C) is incorrect since both N and R are reduced. Answer choice (D) is incorrect since G and S are reduced and W is not reduced. Answer choice (E) is incorrect because all three of L, M, and R are reduced. Consequently, answer choice (A) is correct. Of course, one of the most valuable results of answering a List question correctly is that we now know that the hypothetical G-L-M-N-W is a valid solution to the game.
User avatar
 Abhinusa
  • Posts: 2
  • Joined: Jun 08, 2021
|
#87681
Why can G and W both be reduced if s is not present ?
 Robert Carroll
PowerScore Staff
  • PowerScore Staff
  • Posts: 1787
  • Joined: Dec 06, 2013
|
#87753
Abhinusa,

W is the necessary condition of the conditional. Nothing happens if W is reduced; otherwise, that would be the Mistaken Reversal of the statement.

Robert Carroll

Get the most out of your LSAT Prep Plus subscription.

Analyze and track your performance with our Testing and Analytics Package.